conservation of momentum II

The gamma factor has nothing to do with it, the argument holds in non-relativistic electromagnetism too. The energy dumping of the 2q charge alters its velocity in a way different to the energy dumping of the q charge and thus their motion is asymmetric.

So what? The fact is that any non-zero acceleration will mean asymmetric motion between the two particles and the asymmetry is accounted for exactly by the emission energy and momentum of the photons.

You're trying to argue things don't add up right in physics yet you're basing that on a simplification you've made which is designed to make things not add up! Your entire argument is "My simplification leads to invalid predictions" and instead of saying "So my simplification isn't to be trusted too much" you say "Therefore the original full model was wrong!!". Simplifications are used when you aren't interested in exact results, only qualitative or approximate ones. You are making claims about exact models based on simplifications you've made and which are going to, by their very nature, not give exact results.

But if your simplification leads to a model/result which is only 90% accurate then you're incapable of reaching a justified conclusion.

If this 'snake pit' is such a waste of time and you refuse to 'publish' anything here and you have no interest in talking to mainstream lackeys why do you come here? There's plenty of other forums for science discussion, to come to this one and then complain its a waste of your time is just stupid.

Let me guess, it was developed by you. Is that why you wanted to know who to contact to get a Nobel Prize, you think you've got something worthy of it? All talk and no peer review makes Uno Hoo full of BS.

I have never had any interest in a Nobel Prize. My posts re that subject were a subtle attempt to make you see what an ass you were being. Subtlety can only be overlooked by such an idiot as you. You were complaining that I was bothering you HERE instead of applying for the Nobel. I wanted to show you your stupidity by letting you discover that the Nobel process is virtually denied to anyone who is not already an ass kissing member of the standard party line. You completely missed my point. Just as you completely miss the point of a great many logical things in non-ivory tower life.
 
I have never had any interest in a Nobel Prize. My posts re that subject were a subtle attempt to make you see what an ass you were being. Subtlety can only be overlooked by such an idiot as you. You were complaining that I was bothering you HERE instead of applying for the Nobel. I wanted to show you your stupidity by letting you discover that the Nobel process is virtually denied to anyone who is not already an ass kissing member of the standard party line. You completely missed my point. Just as you completely miss the point of a great many logical things in non-ivory tower life.
See, you've spent more time complaining how I'm in the wrong than it would have taken you to provide a link to a reputable reference showing you're right and I'm in the wrong. It seems you've learnt nothing since the incidents Pete mentions, you still make claims without any justification or back up. I know you want to believe there's some mass conspiracy of silence keeping you down, that because you're not 'an ass kissing member of the standard party line' then there's a discrimination against you but the clear fact is you fail to meet even the most basic standards of even an informed discussion, never mind scientific research. I've repeatedly asked you to provide citations for your claims in this thread and you've given pathetic excuses like I'm not worth the time. The fact you then go on to spend more time taking more pot shots at me displays your deception. As Pete's comments show, you've been like this for years and thus the reason you're an abject failure at physics falls squarely at your own feet. Every single hack thinks that there's a conspiracy of silence against them, never entertaining the thought that perhaps their complete ignorance and lack of intellectual honesty is to blame.

Most people learn that the 'Its everyone else's fault but mine' attitude is a naive and childish excuse for failure pretty early in life. How did you get to 67 and not learn that?
 
  • CANGAS was banned temporarily, not permanently.
  • You've been talking about your archives of my claim that momentum is not conserved since at least [post=1422431]March 2007[/post]. Despite [post=2059320]repeated requests[/post], you've never produced any evidence of your claim. That's why you were temporarily banned by James, as he [post=1432451]clearly stated[/post].
  • You appear to think that I have engaged in wholesale editing and revamping of threads in which we were involved. You are mistaken. When I was moderator, I did, on occasion, edit posts and threads. However, I never did so without leaving a clear trail - a post explaining what was edited and why.

I was following the exchanges between you and CANGAS with unrelenting interest and with daily sequential archive copies. It began in the Trillerian thread "Mass Does Not Increase With Velocity" or some close paraphrase. I am typing this without wasting the effort to look it up because I don't really give a shit about you, your lying, or this Bullshit forum. It does nettle me when a supposedly respectable public institution betrays it trust in such a blatantly dishonest and crooked way. CANGAS raised the issue of how Tr...'s theory violated conservation of momentum. You blatantly argued that momentum is often NOT conserved in Relativity. Then apparently to be a nice guy and not hijack T...'s thread, CANGAS started a new thread, "Mass Does Not Increase With Velocity-DEBUNKED". CANGAS restated the objection that Trillerion's theory violated the conservation of momentum. You, along with Tom2 and others, plainly stated, repeatedly, that Relativity violates momentum and nobody worries about it. You said that the CANGAS thread would be thrown in the shit hole. CANGAS demanded that the Ttil... thread also be shit canned. You repeatedly said that T.... would be protected but not CANGAS.

CANGAS started the thread that is the subject here. In it he plainly explained that momentum is sacred in Physics and you said it is nothing to worry about. You made a considerable number of posts in which you plainly said that Relativity violates momentum. Then suddenly the next day your posts had vanished and were replaced with posts which said that momentum is sacred in agreement with CANGAS. Then CANGAS was banned for "offending a member".

NOW, we see in the locked thread, 98% of both CANGAS posts and Pete goofy anti-momentum posts have been "vanished", as CIA goons say. And the anti-momentum posts of other forum sock puppets.

I don't know exactly who has performed such an outrageous breach of public trust. Pete had his hand on the posts, being a moderator at the time of these fiascos. And Pete is obviously sensitive to peer opinion, since he is such a duck out of water. An MB among a gaggle of Super-Bitch PhDs.

All I know is that CANGAS was seemingly just honestly trying to argue in favor of mainstream Physics beliefs but was blindsided and sucker punched by a gang of self serving yahoos.

I don't give a shit if you jerks ban me, because I, and everyone else who followed the charade have already banned this CRAP site from serious consideration. What happened to CANGAS could happen to any and every honest scientist who disagrees with one of you Bitches. You know that you will never have the talent to do anything worthwhile in Physics and you are trolling for any victim you can overpower.
 
I don't give a shit if you jerks ban me, because I, and everyone else who followed the charade have already banned this CRAP site from serious consideration. What happened to CANGAS could happen to any and every honest scientist who disagrees with one of you Bitches. You know that you will never have the talent to do anything worthwhile in Physics and you are trolling for any victim you can overpower.
Don't let the door hit you on the way out then. I've said repeatedly, if this play is such a 'snake pit' and full of 'jerks' and 'ass kissers for the standard party line' and you've decided nothing will be 'published' here why are you still here? You spend more time giving excuses about why you won't justify your claims then it would take to justify your claims. Says it all.

You said nothing will be 'published' here but you won't publishing anything anywhere reputable. Feel free to prove me wrong.
 
Cangas, I don't know if you are deluded or lying, but your reconstruction of events is more fantasy than fact, and it is boring in its repetition.

I bent over backward to be fair toward you when I was moderator. So much so that two valuable and knowledgeable posters, Tom2 and Trilarian, left because I wouldn't moderate your bullshit more heavily. And unlike you, I can provide evidence.

Don't give a shit? Then stop visiting Sciforums.
 
Uno Hoo:

I am typing this without wasting the effort to look it up because I don't really give a shit about you, your lying, or this Bullshit forum.

Then leave. You followed Pete here in the first place, didn't you?

CANGAS started the thread that is the subject here. In it he plainly explained that momentum is sacred in Physics and you said it is nothing to worry about.

I think I would have remembered if Pete had ever argued that momentum is not conserved in relativity. I would have taken him to task over such a claim.

Then suddenly the next day your posts had vanished and were replaced with posts which said that momentum is sacred in agreement with CANGAS. Then CANGAS was banned for "offending a member".

NOW, we see in the locked thread, 98% of both CANGAS posts and Pete goofy anti-momentum posts have been "vanished", as CIA goons say. And the anti-momentum posts of other forum sock puppets.

Only one post was deleted in the thread linked above, and that was by BentheMan. In particular, none of CANGAS's posts or Pete's were deleted. Any moderator can verify that this is true, from the moderation records.

All I know is that CANGAS was seemingly just honestly trying to argue in favor of mainstream Physics beliefs but was blindsided and sucker punched by a gang of self serving yahoos.

I can't remember the exact sequence of events, but looking at the thread, I banned CANGAS temporarily for 1 month after giving him 1 full week to post evidence for the specific scientific claims he was making. He certainly logged on during the 1 week period, so you can hardly say he was "blindsided" in any way.

I don't give a shit if you jerks ban me, because I, and everyone else who followed the charade have already banned this CRAP site from serious consideration.

Why are you here? Leave. I'll ban you quite happily if you make a direct request and you don't think you will be able to stop yourself coming back for whatever reason. Bear in mind, though, that I'll also ban any new sock puppets you invent. You're lucky that your current one has been allowed to remain here for as long as it has, following the banning of your previous alter-ego.
 
Oh, about Dr. Zion desire for a NEW propulsion. It has been developed. It will be published at the right time and the right place.

By humans?

Are you saying its a super secret technology akin to stealth tech from the 80s? If so, I hardly see any reason.. except maybe the asymmetric colonization of space by different nations.

The gamma factor has nothing to do with it, the argument holds in non-relativistic electromagnetism too. The energy dumping of the 2q charge alters its velocity in a way different to the energy dumping of the q charge and thus their motion is asymmetric.

Actually, the abraham-lorentz force does not mention anything about velocity. Its only acceleration and charge, much like bremsstrahlung.

So, this force will be different on both particles - it will be 4a on 2q and a on q. . .

So we can change the initial to

Ke (2q) = (PE - (4x+4a))
Ke (q) = (PE - (x+a))

But this is hardly elegant .. :(

So what? The fact is that any non-zero acceleration will mean asymmetric motion between the two particles and the asymmetry is accounted for exactly by the emission energy and momentum of the photons.

Which leads to a different point -

If it is all a matter of action and reaction then momentum won't be conserved either (and I'm not considering this very deeply for now). The argument is as follows

Every time a body emits thermal radiation it experiences a recoil, this is what a radiation thruster is based on.

Is this force that you speak of an equivalent to the above, except for emission due to acceleration?

You're trying to argue things don't add up right in physics yet you're basing that on a simplification you've made which is designed to make things not add up! Your entire argument is "My simplification leads to invalid predictions" and instead of saying "So my simplification isn't to be trusted too much" you say "Therefore the original full model was wrong!!". Simplifications are used when you aren't interested in exact results, only qualitative or approximate ones. You are making claims about exact models based on simplifications you've made and which are going to, by their very nature, not give exact results. But if your simplification leads to a model/result which is only 90% accurate then you're incapable of reaching a justified conclusion.

Are talking about the abraham-lorentz ?? I will have to read more about it tomorrow.

I don't know exactly who has performed such an outrageous breach of public trust. Pete had his hand on the posts, being a moderator at the time of these fiascos.

From my experience pete does well arguing hopeless arguments - he has brought me down on my previous conservation of momentum thread and done it without using any dirty tricks.
 
Actually, the abraham-lorentz force does not mention anything about velocity. Its only acceleration and charge, much like bremsstrahlung.

So, this force will be different on both particles - it will be 4a on 2q and a on q. . .

So we can change the initial to

Ke (2q) = (PE - (4x+4a))
Ke (q) = (PE - (x+a))

But this is hardly elegant .. :(
You do realise acceleration and velocity are related right? Both of them, without radiation braking, would feel the same acceleration. With radiation breaking their initial identical acceleration will mean the 2q charge emits more braking radiation and because its braking acceleration it means it doesn't accelerate as much, so it doesn't move as fast. Your formulae are wrong. Surely you should have realised that? I seriously can't understand how you can possibly believe you're coming up with viable equations when you don't know the physics, you admit you don't understand high school mathematics and you've been corrected on just about everything you've said in this thread. Don't you ever learn? If this is your attitude you're going to get nowhere in science.

f it is all a matter of action and reaction then momentum won't be conserved either
You're wrong. These interactions are all basic electromagnetism and electromagnetism conserved energy and momentum. I've told you this several times now. What about it don't you get?

and I'm not considering this very deeply for now
You're not considering it at all.

I will have to read more about it tomorrow.
Don't bother to lie so transparently, you have clearly no interest in learning.

and done it without using any dirty tricks.
Yes, dirty tricks like pointing out your factual errors, your mathematical incompetency and your delusional ignorance. Yes, those dirty tricks are my fault, nothing to do with your inability and unwillingness to accept you're too ignorant to understand.

Stop blaming others for your wilful ignorance, its your fault you keep coming up with crap on a topic you know nothing about.
 
You do realise acceleration and velocity are related right? Both of them, without radiation braking, would feel the same acceleration. With radiation breaking their initial identical acceleration will mean the 2q charge emits more braking radiation and because its braking acceleration it means it doesn't accelerate as much, so it doesn't move as fast.
Right - this is the reason that one will arrive with less kinetic energy - because it doesn't move as fast. But, given the acceleration is sufficiently small 2q and q will still meet halfway, given that acceleration is approaching 0. I do remember this much from high school - that is limits.

The equation will actually look more like this-

Ke (2q) = (PE - (4x+4ax))
Ke (q) = (PE - (x+ax))

Ke (2q+q) = 3x+3ax
P (hv) = 5x/c

I add the x to the a term because the radiation reaction force is proportional to the amount of radiation. It now incorporates energy lost as radiation and energy lost as a result of the radiation reaction force.

Your formulae are wrong. Surely you should have realised that? I seriously can't understand how you can possibly believe you're coming up with viable equations when you don't know the physics, you admit you don't understand high school mathematics and you've been corrected on just about everything you've said in this thread.
By you, for your own purposes. And you are generalizing again.

Don't you ever learn? If this is your attitude you're going to get nowhere in science.
I see you are practicing being a professor already ;)



I have another query if anyone's interested -
Say we only look at 2q from an accelerating reference frame (is this feasible?) which accelerates at the same rate as 2q. We will see that it emits radiation due to the acceleration, right? Because it emits photons it will also have a recoil, right? Will momentum be conserved in this frame alone? I know there is an outside force present, and I'm wondering if an accelerating reference frame would be enough to rule this force out ..
 
Right - this is the reason that one will arrive with less kinetic energy - because it doesn't move as fast. But, given the acceleration is sufficiently small 2q and q will still meet halfway, given that acceleration is approaching 0. I do remember this much from high school - that is limits.
Yes, if you have them move slower and accelerate less then the difference will be reduced but its a simplification. You cannot trust it to give precisely accurate or general results, its a simplification which is only valid for a restricted domain in order to simplify the algebra.

You seem to be saying "In this limiting special case I can simplify things. Now I'll apply that special case everywhere. Oh, I don't get accurate results. Therefore the original full model is wrong!". No, you've simplified something and then been surprised when that simplification isn't able to work everywhere. You're surprised by a tautology!

I add the x to the a term because the radiation reaction force is proportional to the amount of radiation. It now incorporates energy lost as radiation and energy lost as a result of the radiation reaction force.
And you have absolutely no justification in proclaiming anything about the mainstream model if you simply make up your own interpretation of what you think the model says. Its a strawman, you blame the mainstream model for the failure of your simplified guess.

By you, for your own purposes. And you are generalizing again.
Yes, my 'own purposes' are pointing out when someone misrepresents physics, used inconsistent logic and builds strawmen because they're unwilling to accept they can't understand something. Yes, my 'own purposes' are applying the scientific method and being intellectually honest, how terrible!

I see you are practicing being a professor already ;)
Nice attempt to avoid responding to my point. You seem incapable of listening to anything and you delude yourself that the things you simply make up somehow are valid.

Say we only look at 2q from an accelerating reference frame (is this feasible?) which accelerates at the same rate as 2q. We will see that it emits radiation due to the acceleration, right? Because it emits photons it will also have a recoil, right? Will momentum be conserved in this frame alone? I know there is an outside force present, and I'm wondering if an accelerating reference frame would be enough to rule this force out ..
Tell you what, you learn some electromagnetism so we can actually go through the mathematics. Its Catch 22, you won't listen to anything which is mathematically detailed as you don't understand it and you dismiss any wordy explanations as you think you have got something which is qualitatively viable.

If you honestly want to do some kind of science later in life you're going to have to get used to finding information for yourself and putting in some effort. Everyone else has to, you're not special. At least not in the good way.
 
Yes, if you have them move slower and accelerate less then the difference will be reduced but its a simplification. You cannot trust it to give precisely accurate or general results, its a simplification which is only valid for a restricted domain in order to simplify the algebra.

I absolutely can't wrap my mind around this - the acceleration is linked to charge which is linked to potential energy which is linked to bremsstrahlung and the reaction force. The only variables is really mass and distance, but distance doesn't matter so much. Changing the mass will alter acceleration thus bremsstrahlung and the reaction force but it won't alter potential energy or charge.

You seem to be saying "In this limiting special case I can simplify things. Now I'll apply that special case everywhere. Oh, I don't get accurate results. Therefore the original full model is wrong!". No, you've simplified something and then been surprised when that simplification isn't able to work everywhere. You're surprised by a tautology!

This case itself is not special, its a perfectly ordinary scenario.

And to be honest, I can't be sure that the standard bremsstrahlung equation is even relevant in contemporary QED so I definitely don't inted to say the full model is wrong :shrug:

A naive model. Initially both are at rest and feel an attractive force due to Coulomb's law. When released they accelerate towards one another and radiation breaking occurs. This produces a force, which means the net force on each particle is different from if they were say massive particles attracting one another via gravity. To find their velocities you'd need to integrate the relevant equation of motion involving acceleration. The force they feel depends on their separation and their velocity but the velocity is dependent on the force felt over the time since the particles were released. You haven't done this.

To see why this is a problem you can consider the forces the particles feel. Each particle exerts an equal and opposite force on the other, ie if you shut off radiation breaking they'd accelerate symmetrically, at any given time they'd be moving with equal and opposite velocity. If they start at say x=d and x=-d then they'll collide at x=0. Fine. But suppose you could just 'flick a switch' and turn on radiation breaking at some point during this attractive movement. It is as you say, at that instant the 2q particle would have a power output via radiation 4 times than the q particle. What you've done is assume that this is true throughout, so if you had breaking from the beginning then over the time till they collide the 2q particle will emit 4 times as much energy. Wrong. For a given velocity the 2q particle emits 4 times the energy per unit time than the q particle. But since the 2q is emitting more breaking radiation than the q particle it will feel a different net force than the q particle and thus will experience a different acceleration. It'll not be moving at the opposite velocity to the q particle for any time other than when you first 'flick the switch'. It'll emit more so it'll not be as fast and thus if 2q starts at x=-d and q at x=d then they'll collide at some x=D < 0 with the q particle moving faster than the 2q particle. This doesn't have zero momentum total, unlike the initial state, but that's to be expected as the 2q particle has emitted more photons than the q particle has and they carry momentum.

It is, however, done for the non-relativistic case here. That page also mentions how this is precisely the kind of thing which is measured in QED and is the most accurate prediction EVER.

The power output due to radiation breaking depends on the charge and the acceleration, but the acceleration itself depends on the force exerted due to that output of photons! This interdependency is non-trivial and so your assumption the 2q emitted 4 times as much is wrong. If all other things (ie acceleration) were equal you'd be right but they aren't all equal. Throughout the 2q particle feels a smaller net force and thus experiences a smaller acceleration. Hence it emits less than 4 times the amount of breaking radiation energy than the q particle does.

This is precisely the kind of mistake you'd avoid if you actually bothered to find out how the models work.

I have admitted this much myself, I appreciate you restating what I have said.

You are right, this interdependency is non-trivial - even with minimal acceleration, something which I have not recognized before. It cant be accounted for using the simple equation I have given.

Wouldn't you agree however that the reaction force will be 4 times greater for 2q than for q ?

Yes, if you have them move slower and accelerate less then the difference will be reduced but its a simplification. You cannot trust it to give precisely accurate or general results, its a simplification which is only valid for a restricted domain in order to simplify the algebra.

Right, but if this domain includes the entirety of possible solutions then you can be certain that something is awry. But also there is canonical momentum and I don't even know what else ... I will have to put in some effort to find out.

A very conservative approximation is that the kinetic energy of the object will be between 1-3 x and the emitted radiation will be 5 x . In this domain you will still have difficulty conserving momentum if you only consider photons and kinetic energy.
 
Last edited:
I absolutely can't wrap my mind around this
That much is obvious

the acceleration is linked to charge which is linked to potential energy which is linked to bremsstrahlung and the reaction force. The only variables is really mass and distance, but distance doesn't matter so much. Changing the mass will alter acceleration thus bremsstrahlung and the reaction force but it won't alter potential energy or charge.
If you knew how to quantify those things using mathematics it'd be a lot clearer. Previously you've made the argument you can do conceptual things and let others worry about the quantitative stuff but this is a demonstration of how that attitude fails. I suggest you learn from this.

This case itself is not special, its a perfectly ordinary scenario.
You said you were considering the case where they move very slowly, as this minimises the accelerations and thus means the amount of energy emitted by the 2q charge is approximately 4 times that of the 1q charge but, as I've explained, this isn't the exact result. In this non-relativistic limit you're able to say "The 4:1 ratio is close to true" but for relativistic setups, where you allow the particles to undergo a lengthy amount of acceleration then you'll find that the ratio drops to less than 4:1 as the braking radiation on the 2q charge means it doesn't accelerate as quickly as it would do without the braking radiation and thus over the time until collision the 2q charge will emit less than 4 times the 1q charge.

And to be honest, I can't be sure that the standard bremsstrahlung equation is even relevant in contemporary QED so I definitely don't inted to say the full model is wrong :shrug:
Please explain how you are in a position to say what is or isn't relevant to contemporary quantum electrodynamics when you don't understand its quantitative structure, you don't understand its qualitative structure, you don't know what relevance these phenomena have to experiments like particle colliders and you don't know any of the courses a student would need to have done to even get onto a quantum field theory course.

You are in a position of total ignorance when it comes to any mainstream physics, especially high level stuff like quantum field theory, so please tell me why you think your comments are in any way substantiated.

You are right, this interdependency is non-trivial - even with minimal acceleration, something which I have not recognized before. It cant be accounted for using the simple equation I have given.
And the reason you didn't recognise it before is because you never looked. You didn't bother to try to find out what the model actually is, you didn't bother to find out about any experiments, you didn't bother to try to understand it at all, you simply plucked a few equations which you didn't understand the origins of (and thus didn't realise you can't apply them blindly to all situations) from Wikipedia.

Wouldn't you agree however that the reaction force will be 4 times greater for 2q than for q ?
For a given acceleration.

To give an example consider the following thought experiment :

Consider a rollercoaster. The cars on them move along a preset path with a preset velocity. If you gave a rollercoaster car a charge Q and then let it go around the track it'd emit some amount of energy (assume this doesn't alter its velocity, the car goes a preset speed regardless), E(Q). Suppose now you do the same thing and you give the car a charge of 2Q. It will emit E(2Q) = 4E(Q). Twice the charge, 4 times the energy emission. That is what you're talking about.

But that isn't how the particles in your example behave, when they emit braking energy they will experience a force which acts against them and thus their overall acceleration will be lower. Each particle has the same mass so for a given force they'll accelerate the same amount. But they don't experience the same forces, the 2q charge will experience more braking and thus will experience a smaller acceleration. The energy output of 2q will be 4 times that of 1q provided their motions are identical but they won't be, the extra braking slows the 2q charge somewhat and it'll emit less energy overall thus it'll emit less than 4 times what the 1q charge does.

I've explained this several times now. Yes, the formula for the energy emission is quadratic in the charge but it also depends on the motion of the particle. The motion of the 2q charge is affected more and it moves slower and thus does not emit as much as it otherwise would if it moves as faster as the 1q charge.

Again, this would be simple for you to grasp if you could do what is basically high school physics. Once again your lack of mathematical ability/knowledge retards the development of your physics understanding.

Right, but if this domain includes the entirety of possible solutions then you can be certain that something is awry. But also there is canonical momentum and I don't even know what else ... I will have to put in some effort to find out.
You explicitly said its only valid for very low speeds, so by your own admission it doesn't apply exactly. And please don't pretend you know about canonical momentum, you can't even understand a Taylor expansion. You're just throwing out buzzwords to try to make it seem like you're well read and putting in effort but its obvious you aren't. When you admit you can't do high school calculus going on to name drop things which aren't encountered until degree level or beyond only further adds weight to the view you're an ignorant hack who isn't above lying.

A very conservative approximation is that the kinetic energy of the object will be between 1-3 x and the emitted radiation will be 5 x . In this domain you will still have difficulty conserving momentum if you only consider photons and kinetic energy.
Well done on getting what I said completely wrong. The emitted energy is less, not more. And you don't 'still' have momentum issues, in electromagnetism, quantum electrodynamics and relativity its conserved at all velocities and configurations. No amount of you stumbling around in the dark butchering equations you don't understand in the hopes they'll say what you want them to say will negate the fact QED, EM and everything else in physics at present have consistent momentum conservation and have had this tested in experiments to the limits of our ability. No model violated momentum conservation in any configuration, no experiment has ever violated it either.

I really can't understand why you're continuing with this. Time after time its demonstrated you haven't got a clue, you don't know the models, you misrepresent mainstream work (which you haven't read), you have no understanding of the scientific method, you argue against reality and when you try to come off as if you understand it but there's just a crossed wire in communication you demonstrate clearly that you don't understand it. Dropping buzzwords only serves to make your ignorance manifest.
 
I really can't understand why you're continuing with this. Time after time its demonstrated you haven't got a clue, you don't know the models, you misrepresent mainstream work (which you haven't read), you have no understanding of the scientific method, you argue against reality and when you try to come off as if you understand it but there's just a crossed wire in communication you demonstrate clearly that you don't understand it. Dropping buzzwords only serves to make your ignorance manifest.

Be cool man, you seems very free all time long with no proper career. You look knowledgeable, please find a proper job other than sitting all time long in front of your desktop computer,or I recommend you to my boss to work at my lab. Be in mind that my organization lab only doing high end science with innovation and new discovery of science.You must think it the critical way,something others haven't discovered yet. Thanks. :D
 
Last edited:
Albert,

Whether or not alphanumeric seems to have to much spare time on his hands, may actually seem more a reflection on your own status. May i advise that before talking about subjects like the one you have with the person you have, that perhaps you can refrain from being hypocrictical.
 
Be cool man, you seems very free all time long with no proper career. You look knowledgeable, please find a proper job other than sitting all time long in front of your desktop computer,or I recommend you to my boss to work at my lab. Be in mind that my organization lab only doing high end science with innovation and new discovery of science.You must think it the critical way,something others haven't discovered yet. Thanks. :D
I typed that reply during my lunch break. I work for a maths research company doing contract work for high end technology companies which they can't solve with their in house teams. This last month I've done stuff related to artificial intelligence and applications of quantum mechanics. I'm certain that I do more high end science in a day than you do in a month. The fact you can't even type a coherent sentence suggests you're a liar about working in a lab. I am employed precisely because I have proven to people with PhDs in high end science that I'm able to think creatively and able to quickly learn areas of high level mathematics and physics.

You and hacks like DRZion are so easy to slap down I literally do it in my spare time. I rattled off that reply to DRZion in about 20 minutes while talking to other people and having lunch.
 
I typed that reply during my lunch break. I work for a maths research company doing contract work for high end technology companies which they can't solve with their in house teams. This last month I've done stuff related to artificial intelligence and applications of quantum mechanics. I'm certain that I do more high end science in a day than you do in a month.

Thats cool.

You and hacks like DRZion are so easy to slap down I literally do it in my spare time. I rattled off that reply to DRZion in about 20 minutes while talking to other people and having lunch.

I was actually going to comment on how shallow and imprecise your posts are, and I thought maybe it was out of pure arrogance!

But now I know that you don't even try to respectfully discuss or apply any of your faculties to this thread or most of the things on these forums.

If you knew how to quantify those things using mathematics it'd be a lot clearer. Previously you've made the argument you can do conceptual things and let others worry about the quantitative stuff but this is a demonstration of how that attitude fails. I suggest you learn from this.

Yeah, I learned i need to use a different approach!

Please explain how you are in a position to say what is or isn't relevant to contemporary quantum electrodynamics when you don't understand its quantitative structure, you don't understand its qualitative structure, you don't know what relevance these phenomena have to experiments like particle colliders and you don't know any of the courses a student would need to have done to even get onto a quantum field theory course.

What I have said is that I might be applying the wrong equation - but this is not likely, since its the absolute simplest scenario you can come up with in regard to bremsstrahlung.

And the reason you didn't recognise it before is because you never looked. You didn't bother to try to find out what the model actually is, you didn't bother to find out about any experiments, you didn't bother to try to understand it at all, you simply plucked a few equations which you didn't understand the origins of (and thus didn't realise you can't apply them blindly to all situations) from Wikipedia.

Its the absolute simplest equation you can find! Anyways, even if I'm right you won't ever recognize it or you wont care if I get support from someone more knowledgable than you, because to you I'm just another hack - or thats what we've been arguing about the past 3 pages.

Consider a rollercoaster. The cars on them move along a preset path with a preset velocity. If you gave a rollercoaster car a charge Q and then let it go around the track it'd emit some amount of energy (assume this doesn't alter its velocity, the car goes a preset speed regardless), E(Q). Suppose now you do the same thing and you give the car a charge of 2Q. It will emit E(2Q) = 4E(Q). Twice the charge, 4 times the energy emission. That is what you're talking about.

But that isn't how the particles in your example behave, when they emit braking energy they will experience a force which acts against them and thus their overall acceleration will be lower. Each particle has the same mass so for a given force they'll accelerate the same amount. But they don't experience the same forces, the 2q charge will experience more braking and thus will experience a smaller acceleration. The energy output of 2q will be 4 times that of 1q provided their motions are identical but they won't be, the extra braking slows the 2q charge somewhat and it'll emit less energy overall thus it'll emit less than 4 times what the 1q charge does.

I've explained this several times now.

As have I, many pages ago. You're not even trying. :rolleyes:

You explicitly said its only valid for very low speeds, so by your own admission it doesn't apply exactly. And please don't pretend you know about canonical momentum, you can't even understand a Taylor expansion. You're just throwing out buzzwords to try to make it seem like you're well read and putting in effort but its obvious you aren't. When you admit you can't do high school calculus going on to name drop things which aren't encountered until degree level or beyond only further adds weight to the view you're an ignorant hack who isn't above lying.

Right, I should have said 'momentum releted to a potential in an electric field' to fulfill your expectation of a peasant on the internet. You come here with your degree and you act like youre the smart one! You ACT -
thats all! Your arguments hold no merit!

Well done on getting what I said completely wrong. The emitted energy is less, not more. And you don't 'still' have momentum issues, in electromagnetism, quantum electrodynamics and relativity its conserved at all velocities and configurations. No amount of you stumbling around in the dark butchering equations you don't understand in the hopes they'll say what you want them to say will negate the fact QED, EM and everything else in physics at present have consistent momentum conservation and have had this tested in experiments to the limits of our ability. No model violated momentum conservation in any configuration, no experiment has ever violated it either.

And all reactions, all experiments, all equations show that entropy will not be reversed - but I know the truth ! Theres no stopping this ! I am talking about the physics that were even before our universe happened - they were not the same laws that hold today.

I really can't understand why you're continuing with this. Time after time its demonstrated you haven't got a clue, you don't know the models, you misrepresent mainstream work (which you haven't read), you have no understanding of the scientific method, you argue against reality and when you try to come off as if you understand it but there's just a crossed wire in communication you demonstrate clearly that you don't understand it. Dropping buzzwords only serves to make your ignorance manifest.

AN, I'm hoping you will display some maturity and moral character for once and tell me something about the momentum of electric fields.
 
Last edited:
I was actually going to comment on how shallow and imprecise your posts are, and I thought maybe it was out of pure arrogance!

But now I know that you don't even try to respectfully discuss or apply any of your faculties to this thread or most of the things on these forums.
The fact it takes minimal effort to respond to your posts doesn't mean I'm not trying, it just means your level of conversation is very low.

And your comment about the imprecise nature of my posts is precisely something I've commented on before. You don't know any mathematics, therefore if I provide any mathematics to make my comments precise you'll not understand them. But conversely when I don't bother to put any mathematics into my posts you'll complain that they are too imprecise. You lack the ability to discuss precise things and you're blaming me. And you call my attitude into question?!

What I have said is that I might be applying the wrong equation - but this is not likely, since its the absolute simplest scenario you can come up with in regard to bremsstrahlung.
A simplified scenario will not be the most general. Simple doesn't equal widely applicable. Newtonian mechanics is simpler than relativity but you can't use Newtonian mechanics for high velocities. You'd know this if you'd done any physics.

Its the absolute simplest equation you can find! Anyways, even if I'm right you won't ever recognize it or you wont care if I get support from someone more knowledgable than you, because to you I'm just another hack - or thats what we've been arguing about the past 3 pages.
Excellent, you demonstrate basic misunderstandings and then in the same paragraph blame me!

As have I, many pages ago. You're not even trying. :rolleyes:
I'm not even trying because you're not even providing any conversation where I need to concentrate to reply. When I type very detailed or mathematical posts I'll stop listening to music or pause any video I might have playing. I type replies to you during lunch while engaging in conversations with people in the office. Its not my fault you fall at the first hurdle.

Right, I should have said 'momentum releted to a potential in an electric field' to fulfill your expectation of a peasant on the internet. You come here with your degree and you act like youre the smart one! You ACT - thats all! Your arguments hold no merit!
My arguments hold merit, you just don't want to hear them. I've explained why your assumption the energy emitted by 2q charge is always 4 times that of the 1q charge is wrong. If you had it within your grasp to be able to do the equations you'd understand why you're mistaken. Your fault, not mine. If you could write down the equations of motion you'd see I'm right, but your lack of mathematical ability means you can't and now you're blaming me. Take some damn responsibility!

And all reactions, all experiments, all equations show that entropy will not be reversed - but I know the truth ! Theres no stopping this ! I am talking about the physics that were even before our universe happened - they were not the same laws that hold today.
Now you're just going off on a tangent.

AN, I'm hoping you will display some maturity and moral character for once and tell me something about the momentum of electric fields.
So I'm lacking maturity because you refuse to do any learning yourself and thus I won't spoon feed you? Yes, that's obviously my fault that you're unwilling or incapable of educating yourself in even the most basic of things relevant to your whining. How can I tell you something about the momentum of electric fields if I can't use any mathematics and anything which I say to you which you don't already believe you'll ignore due to lack of precise detail? Its Catch 22, your minimal requirement of detail to accept you're wrong is above the maximum level of detail you're capable of understanding. You have made it such that nothing can convince you you're wrong. Even the fact experiments don't agree with you. You disagree with reality and yet don't accept you're mistaken!
 
Last edited:
I'm not even trying because you're not even providing any conversation where I need to concentrate to reply.

AN, we are standing at the brink of a new era! A time when mankind will explore the stars and colonize new worlds. And all you can do is blame my 'conversation' ?? Put this aside and aid me in my quest.

My arguments hold merit, you just don't want to hear them. I've explained why your assumption the energy emitted by 2q charge is always 4 times that of the 1q charge is wrong. If you had it within your grasp to be able to do the equations you'd understand why you're mistaken. Your fault, not mine. If you could write down the equations of motion you'd see I'm right, but your lack of mathematical ability means you can't and now you're blaming me. Take some damn responsibility!

okay, so maybe its like 3.4 or 3.9 .. so i say the kinetic energy will be something between 1-3x while the photonic energy will be between 3-5 x

these objects can have any amount of mass, and the more massive they are the lower the acceleration, with bremsstrahlung being the square of acceleration - meaning that the scenario will move towards 3x kinetic energy and 5x photonic energy as mass approaches infinity. is this not right?

its a decent approximation

it gets messy when a reaction has feedback on the action which produces it, but there is a limit to this endless cycle - i remember this much from 9th grade analysis
 
AN, we are standing at the brink of a new era! A time when mankind will explore the stars and colonize new worlds. And all you can do is blame my 'conversation' ?? Put this aside and aid me in my quest.
Aid yourself. Why should I put in effort to help you when you're not willing to put in effort to help you? If you don't think you're worth the time then I don't. As for colonising the new worlds, do you honestly think you'll help with that if you continue down the road you're on? As it happens, thanks to my job I will be contributing to that goal in a small way.

okay, so maybe its like 3.4 or 3.9 .. so i say the kinetic energy will be something between 1-3x while the photonic energy will be between 3-5 x

these objects can have any amount of mass, and the more massive they are the lower the acceleration, with bremsstrahlung being the square of acceleration - meaning that the scenario will move towards 3x kinetic energy and 5x photonic energy as mass approaches infinity. is this not right?
The energy put into the photons goes down as mass goes up and/or acceleration goes down. Not rocket science.

its a decent approximation
And you're a good judge of what is decent because.....?

it gets messy when a reaction has feedback on the action which produces it, but there is a limit to this endless cycle - i remember this much from 9th grade analysis
That 'messy' bit is precisely what you're making claims about and you're assuming non-messy things in order to grasp the messy bits. Whenever you make an approximation or take a limit you're going to get a non-exact answer. Your entire point in this thread has been to claim there's something that photons can't account for because there's a slight difference between what is predicted and what you think happens. Given you don't understand what is predicted exactly or what happens exactly your entire position is baseless.
 
Back
Top